Đến nội dung

Diepnguyencva

Diepnguyencva

Đăng ký: 01-09-2017
Offline Đăng nhập: 30-07-2019 - 21:16
**---

#720640 $x^{2}-3x+3=(4+3x-\frac{4}{x})\sqrt{x-1}$

Gửi bởi Diepnguyencva trong 04-03-2019 - 19:45

1) $x^{2}-3x+3=(4+3x-\frac{4}{x})\sqrt{x-1}$

2) $x^{2}\sqrt[4]{2-x^{^{4}}}-1=x^{4}-x^{3}$




#719808 BẤT ĐẲNG THỨC

Gửi bởi Diepnguyencva trong 30-01-2019 - 21:29

Cho các số $a,b,c > 0$ và $a+b+c=1$. Tìm giá trị nhỏ nhất của:

 

$ P= \frac {a^2}{b}+\frac {b^2}{c}+\frac {c^2}{a}+\frac {1}{a^2+b^2+c^2}$

$(a+b+c)(a^{2}+b^{2}+c^{2})=(a^{3}+ab^{2})+(b^{3}+bc^{2})+(c^{3}+ca^{2})+(a^{2}b+b^{2}c+c^{2}a)\geq 3(a^{2}b+b^{2}c+c^{2}a) \Rightarrow a^{2}b+b^{2}c+c^{2}a\leq \frac{a^{2}+b^{2}+c^{2}}{3}$

Mặt khác, P$\geq \frac{(a^{2}+b^{2}+c^{2})^{^{2}}}{a^{2}b+b^{2}c+c^{2}a}+\frac{1}{a^{2}+b^{2}+c^{2}}\geq 3(a^{2}+b^{2}+c^{2})+\frac{1}{a^{2}+b^{2}+c^{2}}$




#719807 Đề thi HSG toán 9 tỉnh Hải Dương năm 2018-2019

Gửi bởi Diepnguyencva trong 30-01-2019 - 20:50

de-thi-chon-hoc-sinh-gioi-tinh-mon-toan-




#719695 giải hệ phương trình

Gửi bởi Diepnguyencva trong 24-01-2019 - 20:34

$\left\{\begin{matrix} x\sqrt{y-1}+y\sqrt{x-1}=1 & \\ x^{2}y^{2}+16x+16y=12+20xy& \end{matrix}\right.$




#719570 giải hệ phương trình

Gửi bởi Diepnguyencva trong 18-01-2019 - 19:01

Giải hệ phương trình:

$\left\{\begin{matrix} x^{3}+y^{2}=2 & \\ x^{2}+xy+y^{2}=y& \end{matrix}\right.$




#718590 Giải phương trình

Gửi bởi Diepnguyencva trong 21-12-2018 - 21:34

1) Giải phương trình: $8x^{2}+11x+1=(x+1)\sqrt{4x^{2}+6x+5}$




#714983 Giai hệ 3 ẩn

Gửi bởi Diepnguyencva trong 30-08-2018 - 21:44

$\left\{\begin{matrix} x^{2}y=x+z & & \\ y^{2}z=y+x & & \\ z^{2}x=y+z& & \end{matrix}\right.$




#712459 bđt hay

Gửi bởi Diepnguyencva trong 13-07-2018 - 20:58

Cho $(11x+3y-1)^{2}+(5x+2y+1)^{2}\leq 4$

CMR: $\left | x+y+3 \right |\leq \frac{2\sqrt{73}}{7}$




#712017 BĐT thi hsg

Gửi bởi Diepnguyencva trong 05-07-2018 - 21:26

Cho a,b,c>0 thỏa mãn: $\sum \frac{a+b}{c}=2\sum \frac{1}{ab}$

Chứng minh: $\sum a^{2} +3\geq 2\sum ab$




#709623 [TOPIC] HÌNH HỌC ÔN THI VÀO THPT CHUYÊN 2018-2019

Gửi bởi Diepnguyencva trong 30-05-2018 - 21:03

Bài 93: Cho tam giác ABC nhọn và cân tại C. Đường tròn tâm O đường kính BC cắt AB và AC lần lượt tại E và F. Gọi H là giao điểm BF và CE. Đường tròn đường kính EC cắt AC tại M. Gọi K là giao điểm của BM và (O). Chứng minh KC đi qua trung điểm HF.

Bài 94: Từ điểm A nằm ngoài đường tròn (O) kẻ 2 tiếp tuyến AB và AC; cát tuyến ADE sao cho BD< CD; AD< AE. Gọi H là giao điểm của OA và BC. Gọi I là trung điểm DE. Kéo dài IH cắt (O) tại K sao cho H nằm giữa I và K. Gọi S là tâm đường tròn ngoại tiếp tam giác OKA. Chứng minh OS vuông góc IK.




#709527 Min $\sum \frac{a^2(1-2b)}{b}$

Gửi bởi Diepnguyencva trong 29-05-2018 - 20:06

Ta có: $\frac{a^{2}(1-2b)}{b} + b(1-2b)\geq 2a(1-2b)$

Tương tự, VT$\geq \sum 2a(1-2b)-\sum b(1-2b)\geq \sum a+\sum 2a^{2}-4\geq \sqrt{3}-2$




#709215 [TOPIC] HÌNH HỌC ÔN THI VÀO THPT CHUYÊN 2018-2019

Gửi bởi Diepnguyencva trong 24-05-2018 - 21:51

Bài 85: Cho tam giác ABC (AB<AC) nội tiếp (O). Phân giác góc BAC cắt (O) tại D khác A, E đối xứng D qua O. Gọi F là 1 điểm trên cung BD không chứa A,C của (O); FE cắt BC tại G, H thuộc AF sao cho GH song song với AD. Chứng minh HG là phân giác BHC.




#708044 [TOPIC] ÔN THI BẤT ĐẲNG THỨC $\boxed{\text{THPT CHUYÊN}}$...

Gửi bởi Diepnguyencva trong 10-05-2018 - 20:28

Bài 124: Cho a,b,c>0. Chứng minh rằng :

                                 $\frac{a^{3}}{(a+b)^{3}}+ \frac{b^{3}}{(b+c)^{3}}+\frac{c^{3}}{(a+c)^{3}}\geq \frac{3}{8}$

                                                                                     (Đề thi chọn đội tuyển quốc gia dự thi IMO 2005)




#708040 [TOPIC] HÌNH HỌC ÔN THI VÀO THPT CHUYÊN 2018-2019

Gửi bởi Diepnguyencva trong 10-05-2018 - 20:11

Bài 3:Cho đường tròn (O); AB=2R. Lấy C thuộc đường tròn ( AC>BC). Tiếp tuyên tại A và C của (O) cắt nhau tại D, DB cắt (O) tại E. Kẻ CH vuông góc AB. DH cắt AE tại I; CI cắt AD tại K. Lấy F đối xừng với E qua AB.

a, Chừng minh KE là tiếp tuyến của (O)

b, Qua E kẻ đường thẳng song song AB cắt KB tại S . OS cắt AE tại Q. Chứng minh: D,Q,F thẳng hàng.




#707463 bất đẳng thức

Gửi bởi Diepnguyencva trong 01-05-2018 - 15:52

$\frac{1}{a}+\frac{1}{b}+\frac{1}{c}=1\Leftrightarrow ab+bc+ac=abc; \frac{a^{2}}{a+bc}=\frac{a^{3}}{(a+b)(a+c)};$

Áp dụng Cauchy 3 số, $\frac{a^{3}}{(a+b)(a+c)}+\frac{a+b}{8}+\frac{a+c}{8}\geq \frac{a}{2}$